19
$\begingroup$

Question:

Find the integral $$I=\int_{-\frac{\pi}{2}}^{\frac{\pi}{2}}\dfrac{x\sin{x}}{1+\cos^4{x}}dx$$

my try: since $$I=2\int_{0}^{\frac{\pi}{2}}\dfrac{x\sin{x}}{1+\cos^4{x}}dx$$

then I can't.

I know this follow integral $$\int_{0}^{\pi}\dfrac{x\cos{x}}{1+\sin^2{x}}dx=(arcsinh{1})^2-(\arcsin1)^2$$ (this nice integral is sos440 solve it) $$\int_{0}^{\pi}\dfrac{x\sin{x}}{1+\cos^2{x}}dx=(\arcsin{1})^2-0^2$$ (this is very easy integral),because use $$\int_{0}^{\pi}xf(\sin{x})dx=\pi\int_{0}^{\frac{\pi}{2}}f(\sin{x})dx$$

But I can't my problem,Thank you very much!

$\endgroup$

6 Answers 6

7
$\begingroup$

We have $$I = 2 \int_0^{\pi/2} \dfrac{x \sin(x)}{1+\cos^4(x)} dx = 2 \sum_{k=0}^{\infty} (-1)^k\int_0^{\pi/2} x \sin(x) \cos^{4k}(x)dx$$ Now let $\cos(x) =t$. We then get $$I_k = \int_0^{\pi/2} x \sin(x) \cos^{4k}(x)dx = \int_0^1 t^{4k} \arccos(t) dt$$ We now have $$\int t^{4k} \arccos(t) dt = \dfrac{t^{4k+1}}{2(8k^2+6k+1)}(t _2F_1(1/2,2k+1;2k+2;t^2) + 2(2k+1) \arccos(t)) + c$$ Hence, \begin{align} \int_0^1 t^{4k} \arccos(t) dt & = \dfrac1{2(8k^2+6k+1)}(_2F_1(1/2,2k+1;2k+2;1) + 2(2k+1) \arccos(1))\\ & = \dfrac{\sqrt{\pi}}{(4k+1)^2} \dfrac{\Gamma(2k+1)}{\Gamma(2k+1/2)} \end{align} Hence, \begin{align} I & = 2 \sum_{k=0}^{\infty} (-1)^k \dfrac{\sqrt{\pi}}{(4k+1)^2} \dfrac{\Gamma(2k+1)}{\Gamma(2k+1/2)}\\ & = 2 _4F_3(1/4,1/2,1,1;3/4,5/4,5/4;-1)\\ & \approx 1.845096\ldots \end{align} where the last step is nothing but the definition of the appropriate generalized hypergeometric series, i.e., $$_4F_3(1/4,1/2,1,1;3/4,5/4,5/4;z) = \sum_{k=0}^{\infty} z^k \dfrac{\sqrt{\pi}}{(4k+1)^2} \dfrac{\Gamma(2k+1)}{\Gamma(2k+1/2)}$$

$\endgroup$
2
$\begingroup$

$\newcommand{\+}{^{\dagger}}% \newcommand{\angles}[1]{\left\langle #1 \right\rangle}% \newcommand{\braces}[1]{\left\lbrace #1 \right\rbrace}% \newcommand{\bracks}[1]{\left\lbrack #1 \right\rbrack}% \newcommand{\ceil}[1]{\,\left\lceil #1 \right\rceil\,}% \newcommand{\dd}{{\rm d}}% \newcommand{\ds}[1]{\displaystyle{#1}}% \newcommand{\equalby}[1]{{#1 \atop {= \atop \vphantom{\huge A}}}}% \newcommand{\expo}[1]{\,{\rm e}^{#1}\,}% \newcommand{\fermi}{\,{\rm f}}% \newcommand{\floor}[1]{\,\left\lfloor #1 \right\rfloor\,}% \newcommand{\half}{{1 \over 2}}% \newcommand{\ic}{{\rm i}}% \newcommand{\iff}{\Longleftrightarrow} \newcommand{\imp}{\Longrightarrow}% \newcommand{\isdiv}{\,\left.\right\vert\,}% \newcommand{\ket}[1]{\left\vert #1\right\rangle}% \newcommand{\ol}[1]{\overline{#1}}% \newcommand{\pars}[1]{\left( #1 \right)}% \newcommand{\partiald}[3][]{\frac{\partial^{#1} #2}{\partial #3^{#1}}} \newcommand{\pp}{{\cal P}}% \newcommand{\root}[2][]{\,\sqrt[#1]{\,#2\,}\,}% \newcommand{\sech}{\,{\rm sech}}% \newcommand{\sgn}{\,{\rm sgn}}% \newcommand{\totald}[3][]{\frac{{\rm d}^{#1} #2}{{\rm d} #3^{#1}}} \newcommand{\ul}[1]{\underline{#1}}% \newcommand{\verts}[1]{\left\vert\, #1 \,\right\vert}$ $\ds{% I \equiv \int_{-\pi/2}^{\pi/2}{x\sin\pars{x} \over 1 + \cos^{4}\pars{x}}\,\dd x:\ {\large ?}}$

$\large\tt\mbox{Hint:}$ \begin{align} I &= 2\int_{0}^{\pi/2}x\sin\pars{x}\,{1 \over 2\expo{\ic\pi/2}}\bracks{% {1 \over \cos^{2}\pars{x} - \expo{\ic\pi/2}} - {1 \over \cos^{2}\pars{x} + \expo{\ic\pi/2}}}\,\dd x \\[3mm]&= 2\Im\int_{0}^{\pi/2}{x\sin\pars{x} \over \cos^{2}\pars{x} - \expo{\ic\pi/2}}\,\dd x = 2\Im\int_{0}^{\pi/2}{x\sin\pars{x} \over 2\expo{\ic\pi/4}}\bracks{% {1 \over \cos\pars{x} - \expo{\ic\pi/4}} - {1 \over \cos\pars{x} + \expo{\ic\pi/4}}}\,\dd x \\[3mm]&= \Im\int_{0}^{\pi/2}x\sin\pars{x}\,{\root{2} \over 2}\pars{1 - \ic} \braces{2\ic\,\Im\bracks{1 \over \cos\pars{x} - \expo{\ic\pi/4}}}\,\dd x \\[3mm]&= \root{2}\Im\int_{0}^{\pi/2}{x\sin\pars{x} \over \cos\pars{x} - \expo{\ic\pi/4}} \,\dd x \end{align}

G&R-$7^{\ul{\rm a}}$ ed. has an identity $\pars{~{\bf 2.647}.2,\ \mbox{pag.}\ 224~}$ which seems close to this integral but unfortunately it's only valid for $\color{#0000ff}{\large m \not= 1}$: $$ \int{x^{n}\sin\pars{x}\,\dd x \over \bracks{a + b\cos\pars{x}}^{m}} = {x^{n} \over \pars{m - 1}\bracks{a + b\cos\pars{x}}^{m - 1}} - {n \over \pars{m - 1}b}\int{x^{n - 1}\,\dd x \over \bracks{a + b\cos\pars{x}}^{m - 1}} $$

$\endgroup$
1
$\begingroup$

I might be confused, but since $\int \frac{\sin x}{1+\cos^4 x} dx$ is quite easy to integrate (call the integral $F(x),$ it has an arctan and some logs), your integral is easily done by parts, where the answer is $x F(x)\left|_{-\pi/2}^{\pi/2}\right. - \int_{-\pi/2}^{\pi/2} F(x) d x.$

$\endgroup$
8
  • $\begingroup$ You forgot the $x$ before $\sin(x)$. $\endgroup$
    – 0912
    Dec 3, 2013 at 19:19
  • $\begingroup$ @0912 No, I did not. $\endgroup$
    – Igor Rivin
    Dec 3, 2013 at 19:19
  • $\begingroup$ And how do you integrate $F(x)$? $\endgroup$
    – user27126
    Dec 3, 2013 at 19:19
  • $\begingroup$ @Sanchez $F(x)$ is a sum of arctans and logs of linear functions of $x.$ All of them can be integrated by parts. $\endgroup$
    – Igor Rivin
    Dec 3, 2013 at 19:21
  • $\begingroup$ It's not linear functions of $x$, it's a function of $\cos x$ that involves squares and square root. See wolframalpha.com/input/?i=integrate+1%2F%281%2Bx^4%29 $\endgroup$
    – user27126
    Dec 3, 2013 at 19:23
1
$\begingroup$

We can use residue calculus here, although my attempt trades one nasty integral for a slightly simpler one that can be evaluated in terms of hypergeometric functions with a CAS.


Evaluation of $I$

$$\begin{align*} I &= \int_{-\tfrac\pi2}^{\tfrac\pi2} \frac{x \sin x}{1+\cos^4x} \, dx \\ &= \int_{-\infty}^\infty \frac{y \sqrt{y^2+1}}{\left(y^2+1\right)^2+1} \arctan y \, dy \tag1 \\ &= i2\pi \sum_{\operatorname{poles}\in\Gamma} \operatorname{Res} \left[\frac{z \sqrt{z^2+1}}{\left(z^2+1\right)^2+1} \arctan z\right] - \underbrace{\int_1^\infty \frac{r \sqrt{r^2-1}}{\left(r^2-1\right)^2+1} \log\frac{r-1}{r+1} \, dr}_J \tag2 \\ &= i2\pi \left(\frac{e^{-\tfrac{i\pi}4}}2 \arctan\sqrt{-1+i} - \frac{e^{\tfrac{i\pi}4}}2 \arctan\sqrt{-1-i}\right) - J \tag3 \\ &= \frac\pi{2\sqrt2} \left(\pi - \arctan\sqrt{2+2\sqrt2} + \log \sqrt{\frac{1+\sqrt2-\sqrt{2+2\sqrt2}}{1+\sqrt2+\sqrt{2+2\sqrt2}}}\right) - J \end{align*}$$

In the output from $(\star)$ linked below, recall $\arctan x+\arctan\dfrac1x=\dfrac\pi2$ if $x>0$, and observe $$\frac1{\sqrt2}-\frac12 = \frac12 \left(-1+\sqrt2\right) = \frac1{2\left(1+\sqrt2\right)}$$

to simplify the non-hypergeometric term in our result to

$$I = \frac\pi{\sqrt2} \ln\left(2 \sqrt[4]{\frac{1+\sqrt2-\sqrt{2+2\sqrt2}}{1+\sqrt2+\sqrt{2+2\sqrt2}}}\right) + 2 \, {}_4F_3 \left(\left.\begin{array}{c}\frac14,\frac12,1,1 \\ \frac34,\frac54,\frac54\end{array}\right\rvert {-1}\right) + \frac{3\pi}{32\sqrt2} \, {}_4F_3 \left(\left.\begin{array}{c}1,1,\frac54,\frac74 \\ \frac32,2,2\end{array}\right\rvert {-1}\right)$$

We happen to have the identity,

$$\begin{align*} {}_4F_3 \left(\left.\begin{array}{c}1,1,\frac54,\frac74 \\ \frac32,2,2\end{array}\right\rvert {-t}\right) &= \frac{16}{3t}\ln\left(\frac14\sqrt{1+\sqrt{1+t}}\left(\sqrt{1+\sqrt{1+t}}+\sqrt{2}\right)\right) \\[1ex] \implies {}_4F_3 \left(\left.\begin{array}{c}1,1,\frac54,\frac74 \\ \frac32,2,2\end{array}\right\rvert {-1}\right) &= \frac{32}3 \ln\left(\frac12 \sqrt[4]{\frac{1+\sqrt2+\sqrt{2+2\sqrt2}}{1+\sqrt2-\sqrt{2+2\sqrt2}}}\right) \end{align*}$$

so we reduce the result further to a single hypergeometric expression,

$$I = \boxed{2 \, {}_4F_3 \left(\left.\begin{array}{c}\frac14,\frac12,1,1 \\ \frac34,\frac54,\frac54\end{array}\right\rvert {-1}\right)}$$


Massaging $J$

Substituting $t^2=\dfrac{r-1}{r+1}$ then $2u=\dfrac1t-t$ yields a "simpler" integral,

$$\begin{align*} J &= \int_1^\infty \frac{r \sqrt{r^2-1}}{\left(r^2-1\right)^2+1} \log\frac{r-1}{r+1} \, dr \\ &= \int_0^1 \frac{16t^2(t^2+1)}{t^8-4t^6+22t^4-4t^2+1}\log t\,dt \\ &= \int_0^1 \frac{16\left(1+\frac1{t^2}\right)}{t^4+\frac1{t^4} - 4\left(t^2+\frac1{t^2}\right) + 22} \log t \, dt \\ &= 2 \int_0^\infty \frac{\log\left(\sqrt{u^2+1}-u\right)}{u^4+1} \, du \\ &= -2 \int_0^\infty \frac{\operatorname{arsinh} u}{u^4+1} \, du \end{align*}$$

WolframAlpha $(\star)$ can evaluate $J$ and even finds an antiderivative.


Explanation

  • $(1)$ : substitute $y=\tan x$
  • $(2)$ : residue theorem; see below for more details
  • $(3)$ : compute the residues at $-\sqrt{-1-i}=\sqrt[4]{2}\,e^{i\tfrac{5\pi}8}$ and $\sqrt{-1+i}=\sqrt[4]{2}\,e^{i\tfrac{3\pi}8}$ (using the convention $\sqrt z=\sqrt{\left|z\right|}\,e^{\tfrac i2\arg z}$ and $-\pi<\arg z\le \pi$); to simplify, we have

$$\begin{align*} \arctan\sqrt{-1+i} &= \frac i2 \log \frac{1-i\sqrt{-1+i}}{1+i\sqrt{-1+i}} \\ &= \frac i2 \log\left(-2i - 1 - 2\sqrt{-1+i}\right) \\ &= \frac{\pi - \arctan \sqrt{2+2\sqrt2}}2 + i \log\sqrt{1+\sqrt2+\sqrt{2+2\sqrt2}} \\[2ex] \arctan\sqrt{-1\color{red}{-}i} &= \frac{\pi - \arctan \sqrt{2+2\sqrt2}}2 + i \log\sqrt{1+\sqrt2\color{red}{-}\sqrt{2+2\sqrt2}} \end{align*}$$


Applying the residue theorem

Let $f(z)$ be the integrand, swapping $y$ for the complex variable $z$, and let $\Gamma$ denote an indented, semicircular contour. We make a cut along the imaginary axis at $[i,i\infty)$ (as well as along $[-i,-i\infty)$, but that's not totally relevant here). $\Gamma$ is composed of a line segment of length $2R$; a large, broken semicircle centered at the origin of radius $R$; a smaller, broken circle centered at $+i$ of radius $\varepsilon$; and two opposing line segments joining the arcs. Here's a rough sketch:

enter image description here

I claim without proof that the integrals along the circular contours vanish as $R\to\infty$ and as $\varepsilon\to0$.

On our selected branch we have

$$\begin{align*} \sqrt{z^2+1} &= \sqrt{\left|z^2+1\right|} e^{\tfrac i2 \arg(z^2+1)} \\[1ex] \arctan z &= \frac i2 \log \frac{1-iz}{1+iz} \\ &= \frac i2 \left(\log\left|\frac{1-iz}{1+iz}\right| + i \arg\frac{1-iz}{1+iz}\right) \end{align*}$$

Both $\sqrt{z^2+1}$ and $\arctan z$ behave similarly to either side of the cut in the sense that the argument term for both component functions approaches $-\pi$ from the left/outward bank and $+\pi$ from the right/inward bank. In summary,

$$\begin{array}{|c|c|c|} \hline \rm bank & \rm path & f(z) \\ \hline \rm outward & \begin{cases}z = ir e^{i\phi} \\ dz = i e^{i\phi} \, dr \\ r\in[1+\varepsilon,R]\end{cases} & \displaystyle \frac{z\,e^{-\tfrac{i\pi}2}\,\sqrt{\left|z^2+1\right|}}{\left(z^2+1\right)^2+1} \cdot \frac i2 \left(\log\left|\frac{1-iz}{1+iz}\right| - \frac{i\pi}2\right) \\ \hline \rm inward & \begin{cases}z = ir e^{-i\phi} \\ dz = ie^{-i\phi} \, dr \\ r\in[R,1+\varepsilon]\end{cases} & \displaystyle \frac{z \, e^{\tfrac{i\pi}2} \, \sqrt{\left|z^2+1\right|}}{\left(z^2+1\right)^2+1} \cdot \frac i2 \left(\log\left|\frac{1-iz}{1+iz}\right| + \frac{i\pi}2\right) \\ \hline \end{array}$$

As $\phi\to0$, the integrals along the banks converge to, and provide a total contribution of,

$$\begin{align*} \int_{\rm out} f(z) \, dz &= -\frac12 \int_1^\infty \frac{r \sqrt{r^2-1}}{\left(r^2-1\right)^2+1} \left(\log\frac{r+1}{r-1} - i\pi\right) \, dr \\[2ex] \int_{\rm in} f(z) \, dz &= -\frac12 \int_1^\infty \frac{r \sqrt{r^2-1}}{\left(r^2-1\right)^2+1} \left(\log\frac{r+1}{r-1} + i\pi\right) \, dr \\[2ex] \implies \int_{\rm net} f(z) \, dz &= \int_1^\infty \frac{r \sqrt{r^2-1}}{\left(r^2-1\right)^2+1} \log\frac{r-1}{r+1} \, dr \\ \end{align*}$$

$\endgroup$
1
  • $\begingroup$ Could you find 1.845...? $\endgroup$
    – Bob Dobbs
    Jun 8, 2023 at 18:27
0
$\begingroup$

I am not sure if this integral has a closed solution. I used both Wolfram and Sage to calculate the indefinite integral, $\int\frac{\sin{x}}{1+\cos^{4}{x}}$ but both failed. However you can use a numerical method to get an approximate answer to a precision of your liking. Sage gives the approximate answer 1.8450963514045045.

$\endgroup$
3
  • $\begingroup$ Mathematica gives answer as a generalized hypergeometric function: 2HypergeometricPFQ[{1/4, 1/2, 1, 1}, {3/4, 5/4, 5/4}, -1]. $\endgroup$
    – 0912
    Dec 3, 2013 at 18:59
  • $\begingroup$ @0912, goo.gl/LrP9Wh $\endgroup$ Dec 3, 2013 at 19:01
  • $\begingroup$ @lab bhattacharjee, Yes? Looks like Wolframalpha can't compute it. It exceeds the computation time. $\endgroup$
    – 0912
    Dec 3, 2013 at 19:15
0
$\begingroup$

$$2\;_4F_3[1/4,1/2,1,1;3/4,5/4,5/4;-1]=2\times0.922548...$$. B the substitution $u=\cos x$, expanding the denominator and integrating termwise. I don't think it can be reduced to an elementary expression.

$\endgroup$

You must log in to answer this question.

Not the answer you're looking for? Browse other questions tagged .